ORTHOPEDIC MCQS ONLINE OB 20 RECONSTRUCTION 1D
ORTHOPEDIC MCQS ONLINE OB 20 1D RECONSTRUCTION
-
A 77-year-old man who had right total knee replacement surgery 2½ years ago has had knee pain since surgery. The pain is diffuse, constant, and made worse with activity. He notes warmth and swelling in his knee. Examination shows a well-healed incision, no erythema, moderate warmth, synovitis, and an effusion. The knee is stable, and has an arc of flexion between 3 and 120 degrees. Radiographs show well-fixed and well-aligned implants. What is the most appropriate next step in management?
-
Knee aspiration for culture
-
CT scan of the knee to assess implant rotation
-
Indium, technetium-sulfur colloid scan of the knee
-
Erythrocyte sedimentation rate (ESR) and C-reactive protein (CRP) labs Corrent answer: 4
This patient's history and physical findings are concerning for deep infection. Inflammatory markers (ESR and CRP) should first be obtained, and, if levels are elevated, proceed to knee aspiration for synovial cell count and culture. A bone scan is not indicated in an initial investigation for deep infection, rarely is helpful, and is not cost effective. A CT scan to assess implant rotation is an appropriate investigation for knee pain when the clinical scenario is not suspicious for a deep infection and when infection has been excluded.
-
-
Figures 15a and 15b are the 6-week postsurgical anteroposterior hip radiograph and current radiograph of a 54-year-old avid hiker who returns for routine follow-up 3 years after an uncomplicated uncemented modular metal-on-metal hip replacement. He reports mild activity-related aching diffusely around the right hip region, but does not feel restricted with his activities. Examination reveals no local tenderness, a well-healed incision, and mild discomfort at the extremes of rotation. An erythrocyte sedimentation rate (ESR) and C-reactive protein (CRP) are obtained, showing 9 mm/h (reference range, 0-20 mm/h) and 2.0 mg/L (reference range, 0.08-3.1 mg/L), respectively. What is the etiology of the radiographic finding?
-
Osteolysis secondary to metal particle wear (an adverse reaction to metal debris)
-
Osteolysis secondary to loosening of the femoral implant
-
Metastatic lesion to the proximal femur
-
Deep periprosthetic joint infection Corrent answer: 1
The radiograph shows a large area of osteolysis involving the proximal femur. The implants appear solidly fixed on radiographs and, in the absence of symptoms, it is unlikely the implants are loose or infected. Although infection or metastatic disease is a possibility, the normal ESR and CRP values make the diagnosis of deep infection unlikely. The most likely cause for the radiographic findings is wear from metal particles resulting in an adverse local tissue response and osteolysis.
-
-
What risk factor is associated with a poor prognosis after revision of a failed metal-on-metal resurfacing hip arthroplasty to total hip arthroplasty?
-
Femoral neck fracture
-
Osteonecrosis of the femoral head
-
Aseptic loosening of the femoral component
-
Pseudotumor formation
Corrent answer: 4
Recent clinical series have focused on the high complication rates and problems resulting from revision cases with pseudotumor formation. Pseudotumors are probably related to very high levels of cobalt and chrome particulate wear debris. The underlying etiology most likely is edge wear or impingement that destroys the congruity of the articulation. Revision surgery was quite difficult in one series, particularly if there were large amounts of soft-tissue destruction or nerve involvement. There was a high re-revision rate, often resulting from recurrent pseudotumors with large amounts of debris that could not be removed with the initial revision. The other causes of revision that can cause failure of the femoral resurfacing component can be easily revised with a femoral stem component, and results can be nearly as favorable as with a primary hip replacement.
-
-
Ten-year follow-up studies of total hip replacements performed with modern alumina ceramic femoral heads and acetabular liners show what outcomes?
-
Low incidence of osteolysis, squeaking noise, and ceramic head fractures
-
Same incidence of osteolysis as metal-polyethylene total hips of the same design
-
Higher incidence of osteolysis in hips that make audible noises in vivo
-
Elimination of ceramic head fractures resulting from use of improved biomaterials
Corrent answer: 1
Ten-year follow-up data have been published from a number of clinical centers worldwide that describe the outcomes of total hip arthroplasties performed with third-generation alumina ceramic bearings (metal-polyethylene control hips often were included in the same series). These studies show no osteolysis around well-fixed metal components, and a small incidence of revision surgery to address bearing noise (squeaking) or ceramic femoral head fracture. Newer ceramic materials are associated with satisfactory outcomes in terms of elimination of wear-mediated osteolysis, but problems such as bearing noise and catastrophic femoral head failure have not been eliminated even though the risk for these complications is small and much improved compared to risk associated with earlier generations of ceramic bearings.
-
-
What effect does morbid obesity (body mass index [BMI] higher than 40) have on total knee arthroplasty outcomes?
-
No difference in functional outcome
-
Complication rates are similar to those experienced by nonobese patients
-
Revision rates are similar to those experienced by nonobese patients
-
More postoperative radiolucent lines Corrent answer: 4
Patients with a BMI higher than 40 are more likely than nonobese patients to have radiolucent lines on postoperative radiographs. These patients have a higher rate of complications, inferior survivorship with a higher rate of revision, and poorer function scores.
-
-
A 66-year-old woman had fever, chills, and increasing pain in her right hip. She underwent a total hip arthroplasty using large-head metal-on-metal articulation 4 years earlier without complications. Her hip pain began about 1 month ago following several days of productive cough that her primary care physician had diagnosed as a viral illness. She had elevated serology with an erythrocyte sedimentation rate of 70 mm/h (reference range, 0-20 mm/h) and C-reactive protein of 3.5 mg/L (reference range, 0.08-3.1 mg/L). There is no radiographic evidence of loosening or adverse bone remodeling around the hip arthroplasty. What is the most appropriate course of action?
-
Hip aspiration
-
Metal artifact reduction sequence (MARS) MRI
-
Initiate intravenous antibiotics
-
Assess serum metal trace element levels Corrent answer: 1
This patient has a history and laboratory values highly suggestive of infection following her upper-respiratory-tract infection. Hip aspiration is the most appropriate work-up in this acute setting. Metal ion level assessment is not useful in the workup for acute infection. A MARS MRI scan would be less effective in diagnosing infection than hip aspiration. Empirical use of antibiotics is not appropriate.
-
-
When comparing arthroscopic lavage and knee debridement to placebo in patients with chronic symptomatic osteoarthritis, what outcome has been demonstrated?
-
Reliable and durable pain relief
-
No significant benefit for chronic osteoarthritis
-
Up to 75% pain relief for 2 months, then variable response
-
Three-month measurable pain relief, followed by recurrence Corrent answer: 2
Excluding a diagnosis of meniscal tear, loose body, or mechanical derangement, when knee osteoarthritis of indeterminate cause is treated with arthroscopic lavage and debridement, no discernable benefit has been found to offset the risk of surgery. Effects of arthroscopy have not been clinically significant in the vast majority of patient-oriented outcomes measures for pain and function at multiple time points between 1 week and 2 years after surgery.
-
-
Figure 25 is the anteroposterior hip radiograph of a 74-year-old healthy and active man who was seen in the office 18 months after a primary uncemented total hip replacement with a history of 3 hip dislocations. The last dislocation occurred 1 week ago and he was treated in the emergency department with a closed reduction and application of a hip abduction brace. All episodes of dislocation occurred when bending forward. Aside from the episodes of dislocation, his hip functions well. Examination revealed a normal gait with good abductor strength and pain-free hip movement. What is the most appropriate next treatment step?
-
Prescribe physical therapy to work on abductor strengthening and reinforce hip position precautions.
-
Recommend revision of the acetabular component to change cup position and increase the head and liner size.
-
Recommend revision of the head and liner to a larger size using an elevated or oblique liner.
-
Continue use of the hip abduction brace for 6 weeks and follow with physical therapy.
Corrent answer: 2
This patient has had 3 hip dislocations since his hip replacement, and the radiograph shows an underanteverted cup. He will likely continue to dislocate and surgery is indicated. The anteroposterior radiograph indicates that the cup is vertically oriented and not anteverted. His acetabular component is malpositioned and should be revised to provide the highest likelihood for success. A simple head and liner exchange with a malpositioned implant is less likely to succeed. Additionally, the radiograph shows sufficient acetabular bone stock medial to the cup. The addition of acetabular revision caries a low risk for increased morbidity and will allow a large cup with a larger femoral head with sufficient polyethylene thickness.
-
-
A 67-year-old man who underwent an uncomplicated hip arthroplasty 9 years ago has had a 1-week history of groin pain with movement. Radiographs reveal a well-positioned, well-fixed cementless arthroplasty with mild eccentricity of the femoral head within the polyethylene. His serum C-reactive protein (CRP) level is
3.0 mg/L (reference range, 0.08-3.1 mg/L) and erythrocyte sedimentation rate (ESR) is 5 mm/h (reference range, 0-20 mm/h). What is the most appropriate next step in management of the patient?
-
Aspiration of the hip to rule out an infectious process
-
Complete blood count with differential
-
Observation
-
Bone scan
Corrent answer: 3
Both CRP and ESR results are negative, so aspiration of the hip to rule out periprosthetic hip infection is not recommended. The pain may be the result of a noninfectious process such as polyethylene wear with lysis or a muscle strain. A bone scan is of limited value, as is any further bloodwork. If the symptoms continue, further imaging may be of value.
-
-
Cryotherapy has been demonstrated to achieve what effect after total knee replacement?
-
Decreased transfusion requirement
-
Improved pain, swelling, and analgesia
-
Improved range of motion at the time of discharge
-
Better long-term knee range of motion Corrent answer: 3
In a meta-analysis of randomized controlled trials on the efficacy of cryotherapy after total knee arthroplasty, patients treated with cryotherapy had less blood loss but no difference in transfusion requirements. There was better range of motion at the time of discharge from the hospital. There was no improvement in pain, swelling, or analgesia requirements. Patients treated with cryotherapy did not have better long-term range of motion.
-
-
Compared to retention of the native patella in primary total knee arthroplasty, routine patella resurfacing is associated with
-
no patellar complications.
-
an increased occurrence of anterior knee pain.
-
a decreased patellar fracture rate.
-
a decreased risk for revision surgery.
Corrent answer: 4
Despite concerns regarding fracture, osteonecrosis, and patellar clunk, the routine retention of the native patella during primary total knee replacement is associated with a 20% to 30% increased revision risk in large joint registries. In addition, the retention of the native patella results in a 5.7% revision surgery rate in patients with anterior knee pain.
-
-
What clinical outcome is associated with total hip replacements that have metal-metal bearings (compared to total hip replacements with metal-polyethylene bearings)?
-
Soft-tissue sarcomas
-
Similar revision rates at 5 years
-
Increased nephrotoxicity
-
Pseudotumors
Corrent answer: 4
Patients with metal-metal total hip bearings have higher levels of cobalt and chromium in the bloodstream, but systemic migration of wear debris from total hip bearings is also common to total hip arthroplasties with polyethylene bearings. There is no direct evidence that patients with metal-metal total hip arthroplasties experience a higher incidence of cancer. Chromosome abnormalities have been detected in patients with metal-metal hip bearings, and the clinical consequences of this finding remain unknown. Also, pseudotumors can form around the periprosthetic joint space in response to localized metal ion debris and the host inflammatory response, although these tumors are not specific for failed metal-metal total hip arthroplasties. Metal-on-metal hip replacements have higher revision rates compared to conventional hip replacements in multiple registry studies. Although metal-on-metal articulations have not been shown to cause renal failure, they are not recommended in patients with chronic renal insufficiency.
-
-
A 55-year-old man with unilateral osteoarthritis of the hip underwent a total hip arthroplasty using cementless fixation. The acetabular cup was 52 mm and the femoral head was 28 mm and made of cobalt-chromium alloy. The bearing surface was made of annealed highly cross-linked polyethylene, with an estimated thickness of 6.5 mm. What should the orthopaedic surgeon tell the patient regarding wear of the bearing surface?
-
A highly cross-linked polyethylene bearing has superior wear characteristics compared to a conventional polyethylene bearing.
-
A highly cross-linked polyethylene bearing has similar wear characteristics compared to a conventional polyethylene bearing.
-
The incidence of osteolysis is expected to be higher with highly cross-linked polyethylene than with conventional polyethylene.
-
The volumetric wear rate would be lower if a 36-mm femoral head were used.
Corrent answer: 1
In a prospective, randomized clinical trial of 100 patients undergoing cementless total hip arthroplasties, the investigators compared highly crossed-linked polyethylene to conventional polyethylene. All of the femoral heads were 28 mm. The mean follow-up was 6.8 years. The mean head penetration was 0.003 mm/year for the highly cross-linked polyethylene group in comparison to 0.051 mm/year for the conventional polyethylene group (P =
.006). The improved wear is seen with larger-diameter heads as well. The volumetric wear rate of highly cross-linked polyethylene is equivalent to slightly higher with a larger head than a 28-mm head. Incidence of periarticular osteolysis is lower with highly cross-linked polyethylene.
-
-
A 49-year-old active man has groin pain 3 years after undergoing an uneventful total hip replacement using a cobalt-chrome femoral head articulating against a cobalt-chrome acetabular insert. The pain intensifies with activity and travels down his thigh. Examination and radiographic evaluation are not particularly helpful; there is no evidence of spinal or vascular disease. What is the next step in the evaluation of this patient?
-
A 3-phase bone scan
-
Measurement of synovial metal ions levels
-
Erythrocyte sedimentation rate (ESR), C-reactive protein (CRP), and
possible hip aspiration
-
Bearing exchange to a metal-polyethylene combination Corrent answer: 3
ESR, CRP, and possible hip aspiration is the most logical next step even though at some point, bearing exchange may emerge as the ultimate treatment for a metal-metal adverse reaction in this patient. But the initial workup of a patient with a painful total hip that was otherwise functioning well must include the differential diagnosis of infection, which must be excluded with an appropriate laboratory workup, clinical history, and hip aspiration. The latter study may also help to diagnose a reaction to the metal bearing; cobalt and chromium levels in the aspirate can be investigated, and the color and quantity of the aspirate can be examined along with the cell count. Serum levels of metal ions at this stage could be both helpful and difficult to interpret.
-
-
Figures A and B are the radiographs of a 25-year-old woman whose right knee pain has progressed during the last several years to pain with any activity and pain at night. What is the most appropriate treatment?
-
Proximal tibial osteotomy
-
Distal femoral osteotomy
-
Lateral unicompartmental arthroplasty
-
Total knee arthroplasty
-
Arthroscopic partial lateral meniscectomy Corrent answer: 2
This patient is a good candidate for a joint-preserving procedure. Her symptoms and radiographic findings reveal valgus malalignment of the knee with narrowing of the lateral joint space. The alignment can be corrected with a varus-producing distal osteotomy. Most patients do not proceed to knee arthroplasty for at least 10 years after this procedure. Osteotomy is preferred over partial or total knee arthroplasty because of the patient's young age.
Varus proximal tibial osteotomy would result in joint line obliquity.
-
-
An orthopaedic surgeon noticed a displaced calcar fracture during stem insertion when performing total hip arthroplasty using cementless fixation. What is the most appropriate course of action?
-
Intraoperative exploration to determine the extent of the fracture
-
Use of a longer stem without fixation of the calcar fracture
-
Complete insertion of the stem and measures to protect the patient against full weight bearing for 4 weeks
-
Removal of the stem, internal fixation of the fracture, and definitive reconstruction at a later stage after the fracture has healed
Corrent answer: 1
Calcar fractures can occur with both cemented and cementless stem fixation during surgery. The distal extent of the fracture must be identified either by direct visualization or intraoperative radiograph prior to fixation or implantation of the femoral component. The recommended treatment is to fix the calcar fracture with cerclage wires/cables to restore the mechanical stability of the femoral metaphysis. The same stem can be inserted successfully. The majority of these fractures unite without adverse stem fixation problems.
-
-
A 48-year-old woman had an 8-month history of spontaneous onset of left medial knee pain. She was otherwise healthy with an unremarkable past medical history. Prior to the onset of knee pain, she jogged, played tennis, and golfed regularly. She wished to remain active. Examination showed a fit woman with a BMI of 26, a stable left
knee with full range of motion, and some mild medial joint line tenderness. Radiograph results were normal. An MRI scan showed diffuse grade 3 and a focal area of grade 4 chondromalacia on the medial femoral condyle. The medial meniscus had a degenerative signal but no tear. The remainder of the knee showed no additional pathology. What is the most appropriate initial treatment?
-
Lateral heel wedge
-
Low-impact aerobic exercises
-
Glucosamine 1500 mg/day and chondroitin sulfate 800 mg/day
-
Arthroscopic debridement and microfracture of the focal area of grade 4 chondromalacia to reduce risk for progression
Corrent answer: 2
This patient has early medial compartmental osteoarthritis of her knee. According to the 2008 AAOS Clinical Practice Guideline, Treatment of Osteoarthritis of the Knee (Non-Arthroplasty), there is Level 1 evidence and an "A" recommendation for the use of low-impact aerobic exercises. The guideline also has "A" recommendations with Level 1 evidence indicating that glucosamine and chondroitin should not be prescribed and that arthroscopic debridement not be performed in the absence of symptoms of a meniscal tear or loose body. Lateral heal wedge is not appropriate; the AAOS guideline provides a "B" recommendation with Level 2 evidence indicating that a lateral heal wedge not be prescribed.
-
-
Figure 36 is the postoperative photograph of a patient who underwent a total knee arthroplasty 10 days after surgery. Knee aspiration suggests a Streptococcus infection.
-
Stop physical therapy and institute oral antibiotics.
-
Stop physical therapy and institute intravenous (IV) antibiotics.
-
Open irrigation and debridement, polyethylene spacer exchange, and IV antibiotics
-
Remove components and insert an antibiotic spacer.
Corrent answer: 3
An acute postoperative infection during the first 2 to 4 weeks should be treated with a return to the operating room for open irrigation and debridement of the wound. Polyethylene spacer exchange aides in washing out the entire knee joint. IV antibiotics are also indicated in this situation. To address persistent wound drainage, there is no role for oral or IV antibiotics alone. Removal of the arthroplasty components is recommended for infections after the initial 2- to 4-week postoperative period. However, several recent publications demonstrate a failure rate higher than 50% when the organism is a methicillin-resistant Staphylococcus aureus. Six weeks after surgery, this scenario is no longer considered an acute postoperative infection, and most authors recommend a 2-stage protocol with removal of components and placement of an antibiotic-impregnated cement spacer and 4 to 6 weeks of IV antibiotics.
-
-
What is the difference in outcome when comparing high tibial osteotomy (HTO) to total knee arthroplasty (TKA)?
-
TKA has a longer recovery period than HTO.
-
HTO provides more complete pain relief than TKA.
-
HTO is more reliable in older patients than TKA.
-
HTO outcomes among thin, active, young patients who undergo this procedure approach outcomes associated with TKA.
Corrent answer: 4
The ideal candidate for HTO is a thin, active person with a stable knee, unicompartmental knee symptoms, and age younger than 60. TKA offers a shorter recovery period and more complete pain relief than HTO. TKA is believed to be more reliable than HTO for patients older than age 60.
-
-
Figure 39 is a radiograph of a 72-year-old man who underwent an open reduction and internal fixation of a right femoral neck fracture. After 3 months he started to develop pain, and during the next 8 months he complained of progressive pain and shortening of the hip. What is the most appropriate treatment?
-
Girdlestone
-
Total hip replacement
-
Hardware removal
-
Hardware removal with revision open reductions and internal fixation Corrent answer: 2
Even though a relatively short amount of time has passed since the index surgery, this patient has developed significant osteonecrosis that has caused collapse of the bony structures and the hardware prominent. Total hip replacement gives the most efficient pain relief. Hardware removal with or without re-reduction does not provide reliable pain relief. A girdlestone does not allow the patient to function.
-
-
Figure 40 is the radiograph of a 68-year-old woman who has right knee pain that is limiting her activity and severe preoperative valgus deformity. During total knee arthroplasty, what pathologic features are typically encountered?
-
Lateral femoral hypoplasia
-
Internal rotation of the tibia relative to the femur
-
Medial patella tracking
-
Tight medial collateral ligament Corrent answer: 1
In patients with severe valgus deformity, problems frequently encountered include loose or attenuated medial collateral ligament, tight lateral retinaculum and lateral ligamentous structures (lateral collateral, posterolateral corner), atrophic lateral femoral condyle, lateral patella tracking, and external rotation of the tibia relative to the femur. The hypoplastic lateral condyle can cause internal rotation of the anteroposterior cutting block if the posterior condyler line is used for rotational alignment. The medial soft tissues are typically attenuated and stretched.
-
-
A 59-year-old active woman underwent elective total hip replacement using a posterior approach. She had minimal pain and was discharged to home 2 days after surgery. Four weeks later she dislocated her hip while shaving her legs. She underwent a closed reduction in the emergency department. Postreduction radiographs show a reduced hip with well-fixed components in satisfactory alignment. What is the most appropriate management of this condition from this point forward?
-
Observation and patient education regarding hip dislocation precautions
-
Revision to a larger-diameter femoral head
-
Revision to a constrained acetabular component
-
Application of a hip orthosis for 3 months Corrent answer: 1
First-time early dislocations are often successfully treated without revision surgery, especially when there is no component malalignment. In this clinical scenario, it appears the patient would benefit from better education about dislocation precautions. Hip orthoses are of questionable benefit unless the patient is cognitively impaired. Revision surgery can be successful, but is usually reserved for patients with recurrent dislocations.
-
-
Patellar pain, subluxation, or dislocation after total knee arthroplasty can result from which of the following component orientations?
-
Internal rotation of the tibial component
-
Lateralization of the tibial component
-
Lateralization of the femoral component
-
External rotation of the femoral component Corrent answer: 1
Internal rotation of the components of a total knee arthroplasty, both the tibial and femoral components, can lead to symptoms ranging from patellar pain to dislocation. Most researchers agree that proper external rotation of the femoral component is parallel or nearly so to the femoral epicondylar axis with the knee in the 90-degree flexed position. Proper rotational positioning of the tibial component places the midportion of the tibial component rotationally aligned within the medial one-third of the tibial tubercle. Internal rotation of the tibial component causes relative lateralization of the tibial tubercle and the extensor mechanism. Lateralization of the femoral component moves the trochlear groove laterally. Lateralization of the tibial component moves the tibial tubercle medially, which may be beneficial to patellar tracking.
-
-
How does the risk for periprosthetic infection after total knee arthroplasty compare to risk for infection after total hip arthroplasty?
-
Higher in primary arthroplasty
-
Lower in primary arthroplasty
-
Lower in revision arthroplasty
-
Equivalent in both primary and revision arthroplasty Corrent answer: 1
Risk for periprosthetic infection is higher in the knee (1%-2%) than it is in the hip (0.3%-1.3%). The risk for infection is higher after revision joint replacement surgery compared to primary joint replacement surgery.
Osteoarthritis is not associated with a higher risk for periprosthetic infection, but certain inflammatory conditions such as rheumatoid arthritis and psoriatic arthritis place patients at higher risk for postoperative infection.
-
-
What factor is associated with a higher risk for dislocation after total hip arthroplasty?
-
Male gender
-
Previous hip surgery
-
A direct lateral surgical approach
-
Metal-on-metal bearing surfaces Corrent answer: 2
Dislocation after total hip arthroplasty is a multifactorial problem. Numerous risk factors may act independently or cumulatively to increase risk for this complication. Previous hip surgery of any kind is associated with a twofold increased risk for dislocation. Other risk factors include female gender, impaired mental status, inflammatory arthritis, and older age. Numerous studies have shown a lower dislocation rate with a direct lateral approach, although surgical techniques such as capsular repair have significantly lowered the incidence of dislocation after using the posterior approach. Metal-on-metal bearings have been associated with other complications such as adverse tissue reactions but are often used with larger-diameter bearings, which pose lower risk for dislocation.
-
-
What surgical technique has been associated with increased risk for recurrent dislocation after revision total hip arthroplasty?
-
Posterior capsulorrhaphy
-
Use of a jumbo cup
-
Use of a lateralized liner
-
Use of a larger femoral head diameter Corrent answer: 2
When addressing recurrent dislocation after total hip arthroplasty, surgical considerations that must be addressed include approach, soft-tissue tension, component positioning, impingement, head size, and acetabular liner profile. These considerations most often involve tensioning or augmentation of soft tissues, as in capsulorrhaphy or trochanteric advancement; correction of malpositioned components; use of larger femoral head sizes that increase motion before impingement; improving the head-to-neck ratio; and increasing femoral offset. The use of a larger-diameter acetabular component may lead to soft-tissue overgrowth around the liner, causing impingement and increasing the risk for recurrent dislocation.
-
-
A 67-year-old active man returns for routine follow up 12 years after hip replacement. He has no hip pain. Radiographs revealed a
well-circumscribed osteolytic lesion around a single acetabular screw. All hip components were perfectly positioned. Six months later, comparison radiographs show an increase in the size of the osteolytic lesion. A CT scan shows a well-described lesion that is 3 cm at its largest diameter and is localized around 1 screw hole with an eccentric femoral head. What treatment is appropriate, assuming well-fixed cementless total hip components exist?
-
Revision of the polyethylene liner, removal of the screw, and debridement of the osteolytic lesion with or without bone grafting
-
Revision of the acetabular component to a newer design without screws
-
Removal of the screw, revision of the polyethylene liner, and stem cell injection into the lytic lesion
-
Removal of the offending screw from the metal socket and placement of a new polyethylene liner in the existing socket
Corrent answer: 1
With a well-fixed acetabular metal shell and a localized osteolytic lesion, good outcomes can be expected with liner revision in this clinical scenario with retention of the metal socket, assuming no damage to the components or other unexpected findings during revision surgery. Here, complete cup revision is not warranted considering the appropriate implant position. Beaule and associates reviewed 83 consecutive patients (90 hips) in which a well-fixed acetabular component was retained in clinical scenarios such as the one described; no hip showed recurrence or expansion of periacetabular osteolytic lesions. If the metal cup is unstable, or if the osteolytic lesion is not amenable to debridement through the screw hole, acetabular component revision may be indicated.
-
-
What has been identified as a risk factor for total knee arthroplasty failure after previous high tibial osteotomy?
-
Body mass index higher than 35
-
Female gender
-
Preoperative stiffness
-
Advanced age
Corrent answer: 1
Increased weight, male gender, young age at the time of total knee arthroplasty, laxity, and limb malalignment preoperatively have been identified
as risk factors for early failure for total knee arthroplasty following high tibial osteotomy.
-
-
You are caring for an 18-year-old boy with severe hip arthritis and pain from a missed slipped capital femoral epiphysis. You decide that a hip arthrodesis is the best treatment option. What is the optimum position for a hip arthrodesis to maximize function and prevent complications?
-
0° external rotation, 0° adduction, 0° hip flexion
-
5° external rotation, 5° adduction, 20° hip flexion
-
5° external rotation, 15° abduction, 5° hip flexion
-
15° external rotation, 0° adduction, 20° hip flexion
-
15° external rotation, 15° abduction, 5° hip flexion
Corrent answer: 2
Hip arthrodesis is a salvage procedure for patients with hip arthritis without ipsilateral knee, contralateral hip, or lumbar spine pathology. The optimal position for hip arthrodesis is 5 degrees of adduction, 5-10 degrees of external rotation, and 20-35 degrees of hip flexion.
In their review, Beaule et al. discuss the current indications and techniques regarding hip arthrodesis including appropriate leg position, surgical techniques, methods to optimize function, and later conversion to hip arthroplasty.
Callaghan et al. evaluated the long term efficacy (20-25 yrs) of hip arthrodesis. They found the onset of ipsilateral knee, contralateral hip, or lumbar spine pathology usually began 20 years after the arthrodesis. Of their patients, they found a 15% rate of conversion to hip arthroplasty by 20 years.
-
-
What limits indications for the use of constrained liners?
-
Association with periprosthetic fracture
-
Technical difficulty associated with insertion
-
High costs associated with their use
-
High failure rates associated with their use Corrent answer: 4
Because of reports of relatively high failure rates associated with constrained liners, indications are limited to continued instability after appropriate component position or deficient abductor mechanism and instability. Neither cost nor technical insertion issues are relevant with regard to indications for use. Periprosthetic fractures are not associated with constrained liner usage.
-
-
What serum inflammatory marker has the highest correlation with periprosthetic joint infection?
-
C-reactive protein
-
Serum white blood cell count
-
Erythrocyte sedimentation rate
-
Interleukin 6 (IL-6)
Corrent answer: 4
Although CRP and ESR can be elevated in the setting of infection, IL-6 has been shown to have the highest correlation with infection. Serum white blood cell count has been shown to be ineffective in correlating with periprosthetic joint infection.
-
-
A 68-year-old man reports hip pain 15 years after successful cementless total hip arthroplasty. Radiographs show 3 mm of linear wear of the modular acetabular liner and a retro-acetabular osteolytic lesion. Both the titanium femoral and acetabular components appear to be well fixed. The orthopaedic surgeon recommends revision of the acetabular liner and femoral head. This patient is at increased risk for
-
dislocation.
-
periprosthetic fracture.
-
infection.
-
progressive osteolysis.
Corrent answer: 1
Isolated acetabular liner revision is frequently performed in cases of liner wear and periprosthetic osteolysis in the absence of acetabular component loosening. Many reports have documented an increased incidence of dislocation following this type of revision surgery. This dislocation rate can be reduced by using a larger-diameter femoral head at the time of revision. If the
acetabular component is loose or malpositioned, it should be revised. If the locking mechanism is damaged, then a replacement liner may be cemented into the well-fixed shell. Numerous studies have shown that many osteolytic lesions will reduce in size or heal without bone grafting, and removal of the source of wear debris will arrest the progression of osteolysis. The risk for periprosthetic fracture and infection are lower than risk for dislocation in this setting.
-
-
A 61-year-old man with a body mass index of 31 had a 6-month gradual onset of right medial knee pain. Examination revealed a small effusion, stable ligaments, a normally tracking patella, and mild medial joint line tenderness. Standing radiographs show mild medial joint space narrowing. Effective treatment at this stage of early medial compartmental osteoarthritis includes
-
glucosamine 1500 mg/day and chondroitin sulfate 800 mg/day.
-
weight loss through dietary management and low-impact aerobic exercises.
-
arthroscopic debridement and lavage.
-
a valgus-directing brace.
Corrent answer: 2
According to the 2008 AAOS Clinical Practice Guideline, Treatment of Osteoarthritis of the Knee (Non-Arthroplasty), Level 1 evidence confirms that weight loss and exercise benefit patients with knee osteoarthritis. The other responses have either inclusive evidence (a valgus-directing brace) or no evidence to support their use (glucosamine 1500 mg/day and chondroitin sulfate 800 mg/day and arthroscopic debridement and lavage).
-
-
The range of knee mobility after total knee replacement is multifactorial and dependent upon implant design, surgical implantation accuracy, and patient-specific variables. What total knee implant design is associated with the most knee flexion after total knee replacement?
-
Highly conforming articular surface geometry
-
Higher-flexion femoral component design manufactured to allow the most knee flexion
-
Posterior cruciate-stabilized implant, with or without a higher flexion
manufacturing modification
-
Posterior cruciate-retaining design with a mobile bearing, custom implanted based on CT scan data
Corrent answer: 3
A posterior cruciate-stabilized implant has the best support in the literature in terms of the most favorable range of motion after knee arthroplasty, regardless of whether the femoral component is designed with a higher flexion variation. The higher flexion design is a manufacturing variation that is intended to increase motion by clearing the posterior condyles in flexion.
Although the knee may not gain more flexion, this design allows for more safety in deep flexion. The long-term outcomes of increased stresses on the polyethylene are not known, however. By itself, a higher-flexion design does not lead to increased knee mobility. The effects of mobile bearings, custom CT scan-based knee implantation, and highly conforming designs on ultimate knee range of motion are uncertain.
-
-
In total knee arthroplasty, in vitro testing has shown that crosslinking can diminish the rate of polyethylene wear by 30% to 80%.
What other change in material properties is possible when polyethylene is highly cross-linked?
-
Increased ductility
-
Increased wettability
-
Diminished fatigue strength
-
Decreased resistance to abrasive wear Corrent answer: 3
The most important concern regarding highly cross-linked polyethylene relates to decreased mechanical properties. Cross-linking results in reduced ductility, tensile strength, and fatigue crack propagation resistance. These problems have not been identified as causing implant failure in most recent clinical trials, but remain the most important mechanical issues associated with current material processing methods.
-
-
What factor is associated with a high risk for developing pseudotumors after metal-on-metal hip resurfacing?
-
Large-diameter components
-
Age 40 or older for men
-
Age 40 or younger for women
-
Diagnosis of primary osteoarthritis Corrent answer: 3
The recent experience of a large clinical cohort revealed the most likely risk factors as female gender, age younger than 40, small components, and the diagnosis of hip dysplasia causing osteoarthritis. Failure was least likely among men and procedures involving larger components. These data have prompted some authors to caution against use in women and to primarily target candidates who are men younger than age 50. Small components may be more prone to failure because of malpositioning and edge loading, which have been noted to be more common in dysplasia cases.
-
-
A 70-year-old man is scheduled to undergo bearing surface revision for wear and osteolysis 10 years after cementless total hip arthroplasty. The femoral head is 28 mm alumina-oxide ceramic material. The components are in good position, and there is no evidence of fixation loosening of either component by radiograph or preoperative bone scan. What outcome is associated with isolated polyethylene exchange?
-
Reduced risk for future wear and osteolysis with a larger femoral head
-
Reduced risk for future wear and osteolysis with a cobalt chrome femoral head
-
Similar risk for dislocation compared to primary total hip arthroplasty
-
Increased risk for dislocation compared to primary total hip arthroplasty Corrent answer: 4
The major complication associated with polyethylene exchange is postoperative dislocation. Maloney and associates noted a dislocation rate of 11% in a study of 35 hips after such revision. Boucher and associates reported a 25% rate of dislocation in a study of 25 patients. Larger femoral heads result in higher volumetric wear in contrast to smaller-diameter heads. Stem revision is not indicated because there is no fixation loosening. Moreover, stem biomaterial has no effect on polyethylene wear.
-
-
A healthy, active 68-year-old woman had a total hip arthroplasty 3 months ago. She has been to the emergency department with a posterior dislocation 3 times during the last 2 months. Plain radiographs and a CT scan confirm that the acetabular component is oriented in 5 degrees of retroversion and 55 degrees of abduction.
What is the most appropriate treatment?
-
Revision of the femoral and acetabular components
-
Maximizing head-neck ratio and increasing head length
-
Acetabular component revision
-
Closed reduction with an abduction brace and reinforcement of hip precautions
Corrent answer: 3
Acetabular malposition can lead to recurrent instability. When this cause is confirmed, reorientation of the component can lead to successful revision surgery. Revision of the femoral component may not be necessary if the acetabular component is repositioned. Increasing length and maximizing head-neck ratio cannot make up for component malposition. There is no role for nonsurgical treatment in the setting of recurrent instability with component malposition in an active, healthy patient.
-
-
A 55-year-old woman with history of HIV infection is scheduled for revision total knee arthroplasty to address instability. The index surgery was done 3 years ago. What is the white blood cell (WBC) count threshold in the synovial fluid for an infection diagnosis?
1. 100000
2. 25000
3. 2500
4. 250
Corrent answer: 3
Numerous centers have published data supporting the use of synovial fluid WBC level as one of the most accurate diagnostic criterion for infection in total joint arthroplasty. The threshold ranges between 2000 and 3000 WBC/mL. The sensitivity and specificity have been demonstrated to be higher than 90%.
-
What is the most common complication after a total hip replacement done through the anterior (Smith-Peterson) approach?
-
Lateral femoral cutaneous nerve injury
-
Heterotopic ossification
-
Femoral nerve palsy
-
Anterior dislocation
Corrent answer: 1
Responses 2, 3, and 4 can occur after anterior-approach total hip replacement, but their incidence is much lower than that of thigh numbness, which is associated with injury to the lateral femoral cutaneous nerve. This injury is usually clinically inconsequential. Goulding and associates investigated the incidence of lateral femoral cutaneous nerve injury after anterior hip replacement using self-administered questionnaires to assess sensory deficits and various functional scores for up to 1 year after surgery. Among patients, 81% reported some alteration in thigh sensation, but functional scores did not show any limitations. Symptoms also decreased over time and only a small number of patients reported complete resolution. Also, hip injections administered from the anterior direction risk injury to the lateral femoral cutaneous nerve. Patients undergoing anterior total hip replacement should be counseled about this specific risk before surgery.
-
-
Figure 73 is the anteroposterior pelvis radiograph of a 58-year-old woman who reported chronic hip pain and a clunking sensation 18 months after hip surgery. Laboratory test findings are negative for infection. What is the most appropriate treatment?
-
Revision total hip arthroplasty
-
Trochanteric bursa injection
-
Acetabular component revision
-
A course of physical therapy Corrent answer: 1
The radiograph shows a metal-on-metal hip resurfacing arthroplasty. The acetabular component is malpositioned in excessive abduction, which may explain the clunking symptoms and be allowing the joint to subluxate. The best option is to convert this failed surface replacement to a conventional total hip arthroplasty. Metal-on-metal bearings pose risk for synovitis, metallosis, and adverse tissue reactions, particularly when the acetabular component is malpositioned, resulting in edge loading (this patient's situation). Acetabular revision is possible, but would leave the patient with a metal-on-metal bearing and higher risk for failure attributable to a metal debris reaction and a damaged femoral head component.
-
-
Early postoperative infections following primary total hip arthroplasty are most likely caused by which organism?
-
Staphylococcus epidermidis
-
Streptococcus viridans
-
Propionibacterium acnes
-
Staphylococcus aureus
Corrent answer: 4
Staphylococcus aureus is the most common organism cultured in early (fewer than 4 weeks postoperative) periprosthetic infections. Methicillin-resistant Staphylococcus aureus is becoming a more common pathogen in certain patient populations. B-hemolytic Streptococcus and some gram-negative infections can also be found in early postoperative infections. Staphylococcus epidermidis, Streptococcus viridans, and Propionibacterium acnes are more commonly found in late (> 4 weeks postoperative) infections.
-
-
Figures 75a through 75c are the radiographs and CT scan of a 58-year-old woman who underwent cementless left total hip arthroplasty. Nine months after surgery, she continued to have groin pain when she actively flexed her hip. She had trouble walking up stairs and getting out of her car.
-
Trochanteric bursitis
-
Femoral component loosening
-
Iliopsoas tendonitis
-
Acetabular component loosening Corrent answer: 3
There are a number of possible causes of groin pain after total hip replacement, but an exact diagnosis may remain elusive in some patients. Infection should be ruled out with laboratory studies and, if indicated, diagnostic aspiration of the hip joint. Implant loosening should be evaluated by plain radiograph and bone scan, if indicated. Synovitis resulting from wear debris should be considered in patients with polyethylene liners who experience late-onset symptoms, or in any patient with a metal-on-metal bearing. This patient's symptoms are classic for iliopsoas tendonitis. Physical examination usually reveals pain and weakness with resisted hip flexion. A cross-table lateral radiograph and CT scan show that the anterior edge of the acetabulum protrudes beyond the anterior wall, thereby acting as a source of iliopsoas tendon irritation. In such cases, acetabular component revision and repositioning is indicated. Fluoroscopic-guided iliopsoas cortisone injection can help to establish the diagnosis and relieve groin pain. If the acetabular component is well-positioned, then iliopsoas tenotomy should be considered.
-
-
A 72-year-old woman returns 3 weeks after a right total knee replacement. She has been experiencing increasing pain, swelling, and
decreasing range of motion during the last 10 days. Examination shows the knee to be more swollen and warm than what is typical at 3 weeks after surgery. The knee feels stable, but she has diffuse tenderness and range of motion is between 15 and 85 degrees. What is the most appropriate investigation(s) to diagnose the etiology of her current problem?
-
Radiographs of the knee
-
Radiographs, erythrocyte sedimentation rate (ESR), and C-reactive protein (CRP)
-
Radiographs, ESR, CRP, and knee aspiration
-
No investigations are needed; reassure the patient that her findings are typical at this point
Corrent answer: 3
-
-
What is the difference between annealed (below the melting temperature) and remelted highly crossed-linked polyethelyne?
-
Annealing results in lower potential for oxidation in vivo.
-
Annealing results in less change to mechanical properties and strength compared to remelting.
-
Remelting of polyethylene eliminates the potential for oxidation.
-
Remelting of the polyethylene removes the remaining free radicals and makes the polyethylene stronger.
Corrent answer: 2
Polyethylene is remelted to remove free radicals that formed during the radiation process for cross-linking. The disadvantage of remelting polyethylene is that it reduces the mechanical properties of the material. Annealing of polyethylene maintains its mechanical properties but is less effective at removing free radicals, leaving the polyethylene more susceptible to oxidation. However, both annealed and remelted polyethylene have shown in vivo oxidation.
-
-
When discussing metal on metal hip resurfacing versus metal on polyethylene total hip replacement, the surgeon should inform the patient that all of the following are disadvantages of hip resurfacing EXCEPT?
-
Higher dislocation rate
-
Higher periprosthetic fracture rate
-
Increased serum metal ion levels
-
Higher rates of osteonecrosis
-
Larger incision and surgical dissection Corrent answer: 1
Hip resurfacing is associated with higher rates (compared to conventional THA) of AVN, higher serum levels of metal ions, and higher rates of early postoperative fractures, specifically femoral neck fractures. However, the dislocation rate is significantly lower with hip resurfacing due to the larger size of the femoral component and more accurate restoration of anatomic hip biomechanics. Shimmin et al report on hip resurfacing complications, and published dislocation rates are 0.75% at a mean of 3 years’ follow-up. Also, in order to prepare the acetabulum while preserving the femoral head, a more extensive surgical approach and soft tissues releases are required for a resurfacing procedure.
-
-
A 68-year-old woman underwent an uncemented medial/lateral tapered femoral placement during a total hip arthroplasty. The orthopaedic surgeon noticed a nondisplaced vertical fracture in the calcar region of the femoral neck with final implant insertion. What is the most appropriate treatment?
-
Removal of the press-fit implant and cementing of the same femoral stem
-
Removal of the uncemented femoral component and placement of a revision modular taper-fluted femoral stem
-
Removal of the implant, placement of a cerclage cable around the femoral neck above the lesser trochanter, and reinsertion of the implant
-
Final seating of the uncemented femoral component without additional measures
Corrent answer: 3
The recognized treatment for a proximal periprosthetic fracture is to first identify the extent and then optimize the correction. Several studies indicate that proximal cerclage wiring is adequate to create "barrel hoop" stability of the proximal femur. Braided cables offer superior stability compared to twisted wires or Luque wires. Finally, the appropriate postoperative treatment is protected weight bearing for 6 weeks, with periodic radiographs taken at 2-
week intervals. Other options such as cementing the femoral stem and using a revision arthroplasty device are indicated for unstable fractures.
-
-
Figure 94 is the radiograph of a patient who underwent component removal, insertion of an antibiotic spacer, and recent completion of 6 weeks of intravenous antibiotic therapy. The patient's C-reactive protein (CRP) and erythrocyte sedimentation rate (ESR) have decreased and are now within defined limits. The skin is supple and the patient has a range of motion between 10 and 70 degrees.
What is the most appropriate next step?
-
Two weeks off of antibiotics (antibiotic holiday), followed by knee joint aspiration
-
Continued observation for 6 months after surgery to make sure the infection has resolved
-
Joint aspiration for culture and cell count at the time of completion of IV antibiotic therapy
-
Antibiotic spacer exchange Corrent answer: 1
The situation described here is consistent with a successful treatment for deep infection. If the patient's knee aspiration reveals no organisms and the CRP and ESR trend toward normal, the patient is a candidate for a second-stage
revision knee arthroplasty. A 2-week antibiotic holiday is necessary to obtain accurate culture results either with aspiration or soft-tissue specimens at the time of second-stage revision TKA. There is no evidence that waiting for up to 6 months provides a lower recurrence rate, and there may be long-term problems with a spacer left in place for 6 months, such as breakage, instability, or loosening.
-
-
What factor is associated with decreased range of motion to impingement?
-
Skirted modular femoral head
-
Trapezoidal neck geometry
-
Vertical cup inclination of 40 to 55 degrees
-
Anteversion of 10 to 20 degrees of both the stem and cup Corrent answer: 1
The use of modular femoral stems and acetabular implants have increased the number of head, neck, and liner designs, but the features of recent designs can cause intra-articular prosthetic impingement within the arc of motion required to perform daily activities. Impingement may lead to limited motion, increased wear, osteolysis, and subluxation or dislocation. Minimizing impingement involves avoiding skirted heads, matching the femoral head with an appropriate acetabular implant, maximizing the head-to-neck ratio and, when possible, using a chamfered acetabular liner and a trapezoidal, rather than circular, neck cross-section. Computer modeling studies indicate the optimal cup position is 45 to 55 degrees' abduction. Angles < 55 degrees require anteversion of 10 to 20 degrees of both the stem and cup to minimize risk for impingement and dislocation. However, wear or instability may occur with cup inclination exceeding 50 degrees.
-
-
A 68-year-old woman undergoes an uncomplicated total knee replacement through a midline incision that is extended distally to join a previous incision from a high-tibial osteotomy done 12 years previously. Despite relief of pain and appropriate knee mobility at 2 weeks, drainage continues from the distal part of the wound. What are the most appropriate next step(s) in treatment?
-
Oral cephalexin while the wound heals
-
Vacuum suction drain applied over the draining part
-
Intravenous antibiotics and reassess the knee in 24 hours
-
Urgent open debridement of the knee, cultures, and evaluation of inflammatory laboratory data
Corrent answer: 4
Drainage from a knee after arthroplasty can be ominous; here, a previous incision was made for an osteotomy, possibly compromising wound healing. At 2 weeks, persistent wound drainage is not expected and warrants urgent attention with surgical debridement, cultures, and a baseline check of inflammatory laboratory data. Intravenous antibiotics can be started while awaiting cultures, but oral or intravenous antibiotics alone are not sufficient and a vacuum drain is not indicated in this situation.
-
-
Three years after undergoing a metal-on-polyethylene total hip arthroplasty, a 72-year-old woman develops pain with weight bearing and rest. Hip flexion and internal rotation is associated with pain.
Radiographs show no evidence of loosening. What is the most appropriate next step in this evaluation?
-
Bone scan
-
White blood cell (WBC) count
-
Labeled white cell scan
-
Erythrocyte sedimentation rate (ESR) and C-reactive protein (CRP) levels Corrent answer: 4
The ESR is a sensitive serologic indicator of inflammation; however, it is less specific for infection. Other noninfectious conditions may cause an elevated ESR. The CRP is a specific indicator of infection, which may be transiently elevated after uncomplicated total hip arthroplasty, but usually returns to a normal range within 1 to 2 weeks of surgery. If both the ESR and CRP are elevated, there is a high likelihood that a painful hip replacement is infected, and joint aspiration is indicated. Serum WBC is not a reliable indicator of periprosthetic infection, with a sensitivity of only 20%. Bone scan typically demonstrates increased technetium uptake for several years after hip arthroplasty, which is not specific for infection. A labeled white cell scan is useful to rule out infection, but less helpful to detect the presence of infection.
-
-
During total knee replacement with the trial components in place, the knee achieves full extension but experiences tightness in flexion with a range to only 90 degrees. What is the most appropriate action?
-
Resect more proximal tibia
-
Downsize the femoral component
-
Addition of a distal femoral augment
-
Downsize the tibial polyethylene insert
-
Resect more distal femur Corrent answer: 2
This question is stating that a total knee trial is balanced in extension, but tight in flexion. Downsizing the femoral component increases the flexion gap without changing the extension gap. This allows for more flexion without affecting extension. Additional distal femoral resection will create a larger extension gap without affecting the flexion gap. Reducing the thickness of the tibial insert or resecting more tibia would increase both the extension and flexion gaps.
Manson et al review sagittal plane balancing techniques specifically focusing on the differences between posterior condylar referencing guides and anterior femoral referencing guides.
Illustration A depicts the gap changes associated with increasing or decreasing the femoral component size (smaller AP size as seen in Drawing B of the illustration) leads to a larger flexion gap and unchanged extension gap.
-
-
A 62-year-old man undergoes total knee arthroplasty. Preoperative radiographs are shown in Figure A. Following bone resections and placement of trial implants, the knee is stable in flexion, but cannot achieve full extension. Which of the following interventions will most likely result in a knee that is balanced in flexion and extension?
-
Resect more distal femur
-
Resect more distal femur and downsize the femoral component
-
Resect more proximal tibia
-
Decrease polyethelene liner thickness
-
Place posterior femoral augments Corrent answer: 1
The radiograph demonstrates a varus osteoarthritic knee suitable for a standard total knee arthroplasty. In this scenario, the knee is balanced in flexion, but tight in extension. Resecting more distal femur opens up the extension gap without changing the flexion gap resolving the problem.
Resecting more tibia alters both extension and flexion gaps and would cause laxity in flexion. Downsizing the femoral component opens up the flexion gap and would also cause laxity in flexion. Converting to a decreased polyethylene thickness would change both the flexion and extension gaps. Placing posterior femoral augments would tighten the flexion gap and have no effect on the
extension gap.
Please see the excellent table in the review topic link for a full description of sagittal gap balancing for all combinations of flexion/extension mismatch.
-
-
While obtaining informed consent for a lateral closing-wedge tibial osteotomy, what complication should be discussed with the patient as exclusive to this procedure and not encountered in medial opening-wedge tibial osteotomy?
-
Compartment syndrome
-
Plate breakage
-
Neurologic injury
-
Proximal tibiofibular joint disruption Corrent answer: 4
With lateral closing-wedge osteotomy, proximal tibiofibular disruption can occur. This is not seen in medial opening-wedge osteotomy. A technique has been developed to prevent this complication; a fibular osteotomy is performed at the same time as the tibial osteotomy. The other complications listed are seen in both techniques, with nonunion and plate breakage more common in opening-wedge high tibial osteotomy (HTO) and neurologic injury more common in closing-wedge HTO (with issues related to the common peroneal nerve most prevalent). Compartment syndrome is a devastating complication that can occur with any osteotomy, and a high index of suspicion should be maintained during the postsurgical course for patients who develop this condition.
-
-
A 25-year-old wrestler has been experiencing increasing left knee pain since his last professional cage fight. He complains of both pain and instability on the medial side of his left knee. Examination reveals a grade 3 Lachman and pseudolaxity with valgus stress. Dial test findings are normal. Radiographs show medial degenerative changes and 5 degrees of varus alignment. What is the most appropriate treatment?
-
Rehabilitation with vibration-platform weight-bearing squats
-
Anterior cruciate ligament (ACL) reconstruction with autograft bone-tendon-
bone
-
High-tibial osteotomy (HTO)
-
HTO plus ACL reconstruction at the same time Corrent answer: 4
A young athlete with posttraumatic arthrosis of the knee isolated to the medial side poses a challenge. History and examination confirm key findings.
Complaints of both pain and instability warrant concomitant HTO plus an ACL procedure, particularly in young athletes. Rehabilitation with vibration platform is an unproven technique and is not specific to this knee problem. ACL reconstruction or HTO in isolation would not be ideal for this young, active patient.
-
-
A 28-year-old woman underwent a closing-wedge high tibial osteotomy (HTO) for medial compartment overload after medial meniscectomy. Postsurgically, she reported improvement in her medial pain and resumed normal activities. About 9 months after her surgery, however, she reports burning pain in the front of her knee with running. Her examination reveals no joint line tenderness, mild pain with patellar compression, and limited patellar glides. What is the most likely cause of her symptoms?
-
Patella infera (baja)
-
Patella alta
-
Recurrence of medial joint overload
-
Nonunion of the osteotomy Corrent answer: 1
After HTO, particularly in patients who have been immobilized after a closing-wedge osteotomy, patella baja is a common finding. This can precipitate anterior knee pain or patellofemoral pain syndrome. Recurrence of medial joint overload is incorrect because the patient has no medial joint complaints.
Nonunion is less likely with a closing-wedge osteotomy and likely will not result in anterior knee pain.
-
-
A 45-year-old postmenopausal smoker with a body mass index (BMI) of 22 has had severe knee pain for the past year. The pain has been progressing and the patient is now only able to perform activities
of daily living. Knee radiographs reveal medial compartment osteoarthritis without any involvement of the patellofemoral joint or the lateral compartment. What is the contraindication for a high tibial osteotomy (HTO) in this patient?
-
Smoking status
-
Postmenopausal status
-
BMI
-
Radiographic findings
Corrent answer: 1
The principal contraindications to valgus-producing HTO include (1) lateral compartment degenerative joint disease, (2) loss of a significant portion of the lateral meniscus, (3) symptomatic patellofemoral degenerative joint disease,
(4) nonconcordant pain (ie, patellofemoral pain with medial compartment osteoarthritis), (5) smoking, (6) patient unwillingness to accept the anticipated cosmetic appearance of the desired amount of angular correction, and (7) inflammatory arthritis.
-
-
Highly cross-linked ultra high molecular weight polyethylene has which of the following characteristics as compared to conventional polyethylene?
-
Improved ductility
-
Increased fracture toughness
-
Increases the elongation to break
-
Improved resistance to crack propagation
-
Improved wear resistance Corrent answer: 5
Ultra high molecular weight polyethylene (UHMWPE) is a semicrystalline polymer that has been used for over four decades as a bearing surface in total joint replacements. The mechanical properties and wear properties of UHMWPE are of interest with respect to the in vivo performance of UHMWPE joint replacement components. The mechanical properties of the polymer are dependent on both its crystalline and amorphous phases. Altering either phase (i.e., changing overall crystallinity, crystalline morphology, or crosslinking the amorphous phase) can affect the mechanical behavior of the material.
Per the referenced article by Gencur et al, crosslinking improved wear resistance, but also was found to decrease the ability of UHMWPE to resist
crack inception and propagation under cyclic loading. The findings also suggested that annealing as a post-irradiation treatment may be somewhat less detrimental to fracture crack propogation resistance of UHMWPE than remelting. Answers 1 through 4 are inversely false statements.
-
-
Which of the following describes the mechanical axis of a normally aligned limb?
-
A vertical line drawn from the femoral head through the center of the knee down to the center of the ankle
-
A valgus angle of 5-7 degrees created by two lines drawn down the shaft of the femur and tibia
-
A varus angle of 3 degrees created by two lines drawn down the shaft of the femur and tibia
-
A vertical line drawn from the femoral head passing 1.5 centimeters lateral to the center of the knee down to the center of the ankle
-
A vertical line drawn from the femoral head passing 1.5 centimeters medial to the center of the knee down to the center of the ankle
Corrent answer: 1
The mechanical axis is a vertical line that passes through the center of the knee. It should be re-established during total knee arthroplasty. The tibial joint line is normally in 3 degrees of varus with regards to the mechanical axis.
Boldt et al performed a CT study on 38 TKA's knee and found that the posterior condylar axis was within 3 degrees of the transepicondylar axis in 90% of patients.
The Paley article discusses the approach to a limb with deformity. Incorrect Answers:
2: This describes the anatomic axis in a normal knee
3: This describes the anatomic axis of a knee with significant varus deformity 4: This describes the mechanical axis in a knee with valgus deformity
5: This describes the mechanical axis of a knee with varus deformity
Illustration A depicts the mechanical axis of the limb in an anatomic knee, valgus knee, and varus knee (respectively from left to right).
-
-
What is the most accurate description of the relationship between gender and knee loading during landing while playing basketball?
-
Males have greater total valgus knee loading.
-
Females have greater total valgus knee loading.
-
Males have greater total varus knee loading.
-
Females have greater total varus knee loading.
-
There is no gender difference in total varus or valgus knee loading.
Corrent answer: 2
Ford and associates studied 81 high school basketball players and found that females landed with greater total valgus knee loading and a greater maximum valgus knee angle than male athletes. Hewett and associates reported in a study of 205 female athletes that those with increased dynamic valgus and high abduction loads were at increased risk of anterior cruciate ligament injury.
-
-
In total hip arthroplasty, which of the following techniques will lead to improved stability by increasing the abductor tension?
-
Use of a high offset femoral component
-
Decreasing neck length
-
Use of a low offset femoral component
-
Increasing the head size
-
Medializing the acetabular component Corrent answer: 1
Restoration of an appropriate tension to the abductors should be a primary goal in THA and high offset stems are a tool to assist in increasing tension and improving stability in the appropriate patient. A disadvantage of higher offset stems may be lateral prominence and trochanteric bursitis in thin patients.
Incavo et. al demonstrated that potential benefits of a high offset stem included improved joint stability and avoidance of leg lengthening. Stability is improved by increasing soft tissue tension through restoring the abductor moment arm laterally in high offset stems.
Illustration A shows how use of a high offset femoral component increases abductor tension without changing the length of the leg. Illustration B shows a diagram representing the calculation of a joint reaction force through the hip. High offset stems will also decrease the joint reaction force.
-
-
Significant anterior tibial translation occurs during which of the following rehabilitation exercises?
-
Terminal weight-bearing knee extension
-
Terminal non-weight-bearing knee extension
-
Terminal weight-bearing knee flexion
-
Terminal non-weight-bearing knee flexion
-
Mid-range weight-bearing knee flexion Corrent answer: 2
Terminal non-weight-bearing knee extension exercises from 60 degrees to 0 degrees of flexion increase anterior tibial translation. It is for this reason that this type of exercise should be avoided in the early phase of rehabilitation following anterior cruciate ligament reconstruction so as not to place a tensile
strain on the graft. The other rehabilitation exercises either lead to posterior tibial translation in relation to the femur or have no significant effect on tibial translation.
-
-
A 65-year-old female has severe knee arthritis with a significant flexion contracture and valgus deformity. In the recovery room following her total knee replacement, she is unable to dorsiflex her ankle. Management should include?
-
Application of an AFO to prevent an equinus contracture
-
Unwrap any compressive dressings and flex the knee
-
Immediate EMG
-
Open exploration of the peroneal nerve
-
Reassurance
Corrent answer: 2
Immediate treatment of a peroneal nerve palsy following a total knee replacement consists of removing any compressive dressings (which may compress the nerve) and flexing the knee to release tension on the peroneal nerve.
Peroneal nerve palsy is a potentially devastating complication of total knee arthroplasty (TKA) with an incidence between 0.3 to 1.3%. Approximately 50% of patients with peroneal nerve palsies make complete recoveries.
Nercessian et al. discuss the incidence and risk factors associated with postoperative peroneal nerve palsy. They reort risk factors that include correcting a severe valgus deformity (>18 degrees), a flexion contracture (>15.5 degrees), post-operative epidural analgesia, tourniquet time, and postoperative hematoma.
Illustration A shows the peroneal nerve as it wraps around the head of the fibula.
Incorrect Answers:
Answer 1: An AFO offers no utility in the acute setting. Answer 3: EMG offers no utility in the acute setting.
Answer 4: Surgical exploration of the nerve can be considered if no functional recovery is noted after 3-6 months from the onset of injury.
Answer 5: Reassurance would not be appropriate
-
-
A 67-year-old female presents to the emergency department with the distal femur fracture shown in figure A. She undergoes procedure 1 shown in figures B and C. When compared to procedure 2 shown in figures D and E, which of the following is true regarding procedure 1?
-
Increased rate of secondary surgery and nonunion
-
Increased rate of nonunion and malunion
-
Decreased rate of nonunion and malunion
-
Similar rate of nonunion and decreased rate of malunion
-
Similar rate of nonunion and increased rate of malunion Corrent answer: 4
The patient presents with a periprosthetic distal femur fracture above a stable total knee arthroplasty (TKA) implant and undergoes bridge plating. When compared to retrograde intramedullary nailing (RIMN), bridge plating has a similar rate of nonunion and decreased rate of malunion.
Most periprosthetic distal femur fractures have a stable TKA implant. In these cases, internal fixation is recommended and nonsurgical care is reserved for those who are unable to tolerate surgery. Surgical intervention typically relies on orthopaedic principles of relative stability, however simple fracture patterns may be amenable to absolute stability constructs. Anatomically contoured implants and locking technology have made minimally invasive plate osteosynthesis a standard treatment options for these injuries.
Ristevski et al. conducted a systematic review of 719 periprosthetic distal femur fractures. Locked plating and RIMN showed similar rates of nonunion and need for secondary procedures. RIMN had a slightly higher rate of
malunion.
Kolb et al. reported on midterm results of periprosthetic femur fractures treated with the less invasive stabilization system (LISS) locking plate. In 23 patients, they had 2 delayed unions and no nonunions. There was no comparison group.
Figure A is a radiograph showing a periprosthetic distal femur fracture above a total knee arthroplasty. Figures B and C are post operative radiographs after internal fixation with a bridge plate. Figures D and E, in comparison, are post operative radiographs of similar injury after RIMN.
Incorrect Answers:
Answers 1-3,5: Bridge plating has similar rate of nonunion and secondary surgery as RIMN. Bridge plating has a slightly lower rate of malunion when compared to RIMN. The higher rate of malunion of RIMN may be due to constraints in your starting point determined by your implant.
-
-
Ultra-high-molecular-weight polyethylene (UHMWPE) particles have been associated with osteoclastogenesis, a key component of osteolysis in total joint replacement. Which of the following accurately describes the expression of vascular endothelial growth factor (VEGF) and receptor activator of nuclear factor kappa-B ligand (RANKL) during UHWMPE induced osteolysis?
-
RANKL expression is increased and VEGF is unchanged or mildly decreased
-
VEGF expression is increased and RANKL is decreased
-
VEGF expression is increased and RANKL expression is unchanged
-
Expression of both VEGF and RANKL is increased
-
Expression of both VEGF and RANKL is decreased Corrent answer: 4
RANKL, RANK, and VEGF are upregulated in any process that involves increased osteoclast activity.
Ren et al. found in a mouse model study that UHMWPE stimulation significantly increased VEGF, RANK, and RANKL gene expression. VEGF inhibitor was shown to reduce the UHMWPE-stimulated VEGF production. VEGF inhibitor also resulted in reduced RANK and RANKL gene transcripts. Thus, this study suggests that VEGF has a role in the regulation of RANK/RANKL-
mediated osteoclastogenesis.
VEGF is also an important factor in angiogenesis in multiple tissues, including normal tissue (e.g. tendons) and several cancers.
Illustration A depicts the interaction of osteoblasts and osteoclasts (E=estrogen, P-C-P=bisphosphonates, CT=calcitonin). Illustration V is a video tutorial of RANK, RANK-L, and OPG and their role in postmenopausal osteoporosis.
-
-
Figures A and B show a 65-year-old woman with a history of a well functioning right total knee done 5 years prior who presents with pain and inability to bear weight after a fall from standing height. Her midline knee incision is well healed. What is the best treatment at this time?
-
External fixation
-
ORIF with 2.4 mm reconstruction plate
-
Revision of the femoral component with a long stem
-
Antegrade intramedullary nail
-
Retrograde intramedullary nail Corrent answer: 5
The clinical presentation is consistent with a Su Type I periprosthetic fracture with a well-fixed component and cruciate retaining (CR) total knee replacement. Among the options listed the most appropriate treatment is a retrograde intramedullary nail.
Periprosthetic fractures of the distal femur commonly occur following low energy trauma in osteopenic bone. Non-displaced fractures with a stable component may be treated nonoperatively with casting or bracing.
Determination of component loosening versus stability helps guide decision making of revision arthroplasty versus ORIF, respectively. ORIF treatment options most commonly include locked femoral plating or retrograde femoral nailing with additional options such as flexible intramedullary nails, angled blade plates, and dynamic condylar screws. Fracture lines starting at or proximal to the anterior flange of the femoral component (Su Type I/II) and CR knee or posterior stabilized (PS) with an open box design make retrograde femoral nailing a viable option.
Nauth et al. review treatment options for periprosthetic distal femur fractures with a focus on well fixed components. They state advantages of retrograde nailing include minimal soft tissue stripping and ability to use prior midline incision. Disadvantages include difficulty to use in the setting of PS knee with a closed femoral box or ipsilateral total hip arthroplasty, inability to use with limited bone stock or distal fracture line, and potential malreduction.
Gliatis et al. evaluated long term outcomes in 10 periprosthetic distal femur fractures treated with retrograde nailing. All cases went on to union with no statistical difference seen in preoperative and postoperative WOMAC scores.
Kolb et al. followed 23 consecutive patients with periprosthetic distal femur fractures above a stable component treated with locked periarticular plates. All fractures healed at a mean of 14 weeks with minimal complications seen at 46 months.
Images A and B show a distal femur periprosthetic fracture around a well fixed total knee component. Illustration A shows an example of a periprosthetic distal femur fracture treated with retrograde intramedullary nailing. Illustration B shows the Su classification of periprosthetic distal femur fractures.
Illustration C shows the characteristic boxless design of a CR knee replacement. Illustration D shows an outline of the cam or box which is characteristic of a PS knee replacement.
Incorrect Answers:
Answer 1: External fixation would not be the best treatment option in this scenario.
Answer 2: A reconstruction plate would not be appropriate fixation in this scenario. A locked femoral plate would be an option in this scenario.
Answer 3: Revision of the femoral component is not indicated in this scenario as the implant is well fixed.
Answer 4: An antegrade IMN would make adequate fixation distal to the fracture line difficult.
-
-
An 80-year-old female presents following a fall from standing. She was an active, independent, community ambulator prior to this event. Past surgical history is significant for a left total hip arthroplasty 10 years prior. A left hip XR is obtained and shown in Figure A. A CT is obtained and demonstrates a displaced transverse acetabulum fracture with medial cup migration. There is no evidence of femoral component loosening or fracture. There is no concern for infection and all inflammatory markers are within appropriate limits. Which treatment is most appropriate?
-
Restricted weight bearing
-
Acetabular revision with a custom triflange implant
-
Dual approach pelvic ORIF and acetabular revision
-
Acetabular revision with cup-cage construct
-
Acetabular revision with placement of a jumbo cup Corrent answer: 4
Proper management is an acetabular revision to a cup-cage construct, which will provide posterior column fixation.
When evaluating periprosthetic acetabular fractures, the Paprosky
classification can help guide management. This fracture description is similar to a Type IIIB with pelvic discontinuity. A transverse fracture of the acetabulum, by definition, leads to bi-columnar instability. Fixation of the posterior column with a cup-cage implant will address this. Intra-operatively, the femoral head should be exchanged and fixation of the femoral component should be assessed. Finally, in any periprosthetic fracture or dislocation, it is always important to consider and rule out an underlying infection.
Laflamme et al. retrospectively reviewed 32 patients following treatment for intraoperative periprosthetic acetabular fractures. They associated nonunion and displacement with lack of plating displaced posterior column fractures. All anterior fracture patterns in this study healed without incident. The highest rate of complications came with fractures that were not recognized intraoperatively.
Peterson et al. followed 11 patients who sustained an acetabular periprosthetic fracture with 6.2 year mean follow-up. They described type 1 fractures as a clinically and radiographically stable acetabular component and type 2 fractures as unstable. They concluded that displaced fractures often require revision of the acetabular component, but non-displaced fractures can sometimes be treated successfully with weight-bearing restrictions.
Figure A demonstrates protrusio of the acetabular component in a patient status-post left total hip arthroplasty. Illustration A demonstrates a transverse acetabular fracture. Illustration B shows a cup-cage implant. Illustration C shows an XR of a hip revision utilizing a cup-cage implant. Illustration D shows the Paprosky Classification for acetabular bone loss.
Incorrect Answers:
Answer 1: Non-operative care is not indicated in this scenario.
Answer 2: A custom triflange implant is often utilized in cases of massive bone loss and is not needed in this scenario; these implants can also be very expensive.
Answer 3: A bi-columnar approach is not necessary here and adds undue risk to the patient with an intrapelvic approach.
Answer 5: A jumbo cup will not provide adequate support and fixation in the setting of a displaced bicolumnar fracture.
-
-
While trialing components during a routine total knee arthroplasty, the flexion gap is felt to be loose and the extension gap is stable. Which of the following are possible ways to treat this intraoperative instability?
-
Move the femoral component posterior
-
Increase the size of the polyethylene component
-
Downsize the femoral component
-
Move the femoral component anterior and augment the distal femur
-
Externally rotate both the femoral component and tibial components Corrent answer: 1
Flexion and extension gaps during TKA must be balanced to provide a knee that is stable throughout its range of motion. Changes to the tibia, such as modifying the size of the polyethylene insert, affect both the flexion gap and the extension gap. Conversely, changes to the femur typically affect either the flexion gap or the extension gap, but not both. Moving the femoral component anterior or posterior increases or decreases the flexion gap respectively. In a similar manner, moving the femoral component proximally or distally increases or decreases the extension gap respectively. In this question, the patient is loose in flexion and stable in extension. This can be addressed by either moving the femoral component posterior, or upsizing the femoral component which will only increase the AP diameter of the prosthesis. Therefore, answer 1 is correct.
Whiteside et al is a review article that discusses the proper techniques that results in a knee that is balanced to varus and valgus stresses in flexion and extension.
-
-
A 65-year-old active female presents with medial sided knee pain of 2 years duration that has failed nonoperative modalities.
Radiographs are shown in Figure A. Which of the following variables is the strongest contraindication to unicompartmental knee arthroplasty in this patient?
-
BMI greater than or equal to 30
-
Fixed varus deformity of five degrees
-
Five degree flexion contracture
-
Contralateral knee osteoarthritis
-
Rheumatoid arthritis
Corrent answer: 5
While the contraindications for unicompartmental knee arthroplasty (UKA) are controversial and changing, most still consider rheumatoid arthritis to be an absolute contraindication because the pathology affects all compartments of the joint.
UKA for single compartment arthritis has recently become more popular. There is controversy surrounding the indications and contraindications. Relative contraindications include fixed varus deformity of more than 10 degrees, fixed valgus deformity of more than 5 degrees, fixed flexion contracture of more than 3-5 degrees, joint subluxation of 5 mm or greater, and arthritis in more than one knee compartment. Rheumatoid arthritis, on the other hand, is considered an absolute contraindication because the pathology affects all compartments of the joint, which can predispose the patient to an earlier failure following UKA.
Pandit et al. sought to determine whether previously described contraindications to UKA should apply to patients with a mobile-bearing UKA. With regards to BMI, they found no significant clinical or functional outcome difference between 551 UKAs performed in ideal weight patients (44-82kg) compared to non-ideal weight patients (82-185kg).
Argenson et al. evaluated 62 patients (70 knees) at a mean follow-up of 20 years after undergoing UKA. Fourteen of the 70 knees required a revision procedure, with causes including progression of arthritis in other compartments (n=12), polyethylene wear (n=5), aseptic loosening (n=2), and late infection (n=1).
van der List et al. performed a meta-analysis of 31 cohort studies and 6 registries for outcomes of medial UKA patients with respect to traditional exclusion criteria. They found females had inferior outcomes compared to males, and that females and younger patients had a higher likelihood of revision.
Figure A shows an AP radiograph with medial joint space narrowing and sclerosis contained to the medial compartment.
Incorrect Answers:
Answer 1: BMI over 40 is a relative contraindication, but studies including Argenson et al. have not shown any UKA clinical or functional differences when stratified by BMI.
Answer 2: A fixed varus deformity of five degrees is not a contraindication. Generally a varus deformity up to ten degrees is acceptable.
Answer 3: A flexion contracture greater than 5 degrees is a relative contraindication.
Answer 4: Contralateral knee osteoarthritis is not a contraindication to UKA.
-
-
Which of the following complications is the primary reason for early reoperation following the procedure shown in Figure A?
-
Edge loading leading to rapid polyethylene wear
-
Fracture of the femoral neck
-
Pseudotumor formation
-
Infection
-
Groin pain from accelerated acetabular erosion Corrent answer: 2
Fracture of the femoral neck is the primary reason for early reoperation following hip resurfacing procedures.
Hip resurfacing is primarily indicated for younger males with good proximal femoral bone stock. Advocates of hip resurfacing arthroplasty cite preservation of the proximal femoral bone stock as the main advantage of this procedure over total hip arthroplasty. Fracture of the femoral neck has been reported following hip resurfacing. The exact etiology is unknown, but technical errors, such as notching of the femoral neck or disruption of the blood supply to the femoral head during extensive soft-tissue exposure may contribute to femoral neck fracture.
Shimmin et al. provide a review of complications associated with hip resurfacing. Such complications include femoral neck fractures, avascular necrosis, and increased metal ion levels. They also discuss dislocation rates with resurfacing procedures, which are less than total hip arthroplasty procedures.
Amstutz et al. provide a case series of 600 hip resurfacing procedures to evaluate post-operative femoral neck fractures in this setting. Failures were assessed radiographically and with implant retrieval. There were 5 femoral
neck fractures in this series, all following a traumatic event that the authors partially attributed to a failure of covering all of the reamed bone with the metal femoral component.
Figure A is an AP pelvis XR demonstrating a right hip resurfacing procedure. Illustration A shows a femoral neck fracture s/p left hip resurfacing.
Incorrect Answers:
Answer 1: While edge-loading can be a source of accelerated wear (hence why the acetabular component should be placed in less abduction compared to a conventional total hip arthroplasty), there is no polyethylene used in metal-on-metal hip resurfacing.
Answers 3 and 4: Although these are possible reasons necessitating revision of a hip resurfacing, the most common reason for early reoperation remains fracture of the femoral neck.
Answer 5: This is a common cause of revision in a hemi-resurfacing procedure, in which the acetabulum is not resurfaced.
-
-
Which of the following is the most common reason for reoperation following hip resurfacing in the first 6 months following the operation?
-
Aseptic loosening of the acetabular component
-
Aseptic loosening of the femoral component
-
Fracture of the femoral neck
-
Fracture of the acetabulum
-
Infection
Corrent answer: 3
Fracture of the femoral neck is the most common complication and reason for reoperation in the first 6 months following hip resurfacing.
Hip resurfacing is a potential arthroplasty option for select patients. Advocates of resurfacing hip arthroplasty cite preservation of the proximal femoral bone stock, improved stability due to the larger head size, and decreased activity restrictions (patients may run) as advantages. Disadvantages include metallosis due to the metal on metal bearing surface and femoral neck fractures, which are unique to resurfacing. The exact etiology of femoral neck fractures following resurfacing is unknown. Technical errors, such as notching of the femoral neck or possibly disruption of the blood supply to the femoral head during extensive soft-tissue exposure, may result in femoral neck fracture.
Amstutz et al. found that it is important to avoid or at least minimize notching of the femoral neck during hip resurfacing by performing cylindrical reaming at the recommended angle of 140 degrees. Additionally, they note the importance of stopping the reamer before it touches the lateral cortex. Lastly, they recommended that osteophytes should only be removed if there is impingement with the hip is flexed to 90 degrees and internally rotated.
Gabriel et al. report on multiple complications that must be considered with hip resurfacing. Specifically, these complications can include implant loosening, femoral neck fracture, acetabular wear, and persistent pain. There is still controversy on how to treat young patients with large osteonecrotic lesions of the femoral head.
Illustration A is a radiograph of a femoral neck fracture after a hip resurfacing surgery.
Incorrect Answers:
Answers 1,2,4,5: These are not the most common reason for reoperation after hip resurfacing surgery.
-
-
A 40-year-old male presents with chronic severe and progressively worsening right hip pain which has been intractable to conservative management. He has a history of avascular necrosis of the femoral head with subsequent collapse and development of severe osteoarthritis. After discussion of his surgical options, he elects to proceed with the procedure shown in Figure A. He presents to the emergency department 2 months later with severe groin pain and inability to bear weight. Radiographs obtained at this time are shown in Figure B. Each of the following has been shown to increase the risk for development of this complication post-operatively EXCEPT:
-
History of avascular necrosis with cystic bone loss
-
Notching of the superior aspect of the femur
-
Varus placement of the femoral component
-
Incomplete seating of the femoral implant
-
Excessive inclination of the acetabular component Corrent answer: 5
All the variables have been found to increase the risk of femoral neck fracture following hip resurfacing arthroplasty except for excessive inclination of the acetabular component. However this risk is increased with component
malalignment resulting in femoral neck impingement, such as excessive retroversion of the acetabular component.
Hip resurfacing arthroplasty has largely fallen out of favor over the past two decades due to the strong association with metal-on-metal hip-related complications such as metallosis and pseudotumor. However, proponents continue to advocate for its utility in young males with good bone quality due to the purported advantages of reliable functional outcomes, longevity, bone preservation, ease of revision, reduced dislocation risk, and improved stress transfer to the proximal femur thereby minimizing stress shielding. Reliance on the proximal femur for implant stability predisposes the femoral implant to failure when this interface is compromised. This may in turn lead to femoral neck fracture, the most common early complication following hip resurfacing arthroplasty, which has been shown to be linked to factors that weaken this interface. This includes patient-related factors such as poor bone quality, bone loss, large cysts of the femoral head and neck such as in the setting of avascular necrosis, and obesity, as well as technical errors such as notching of the femoral neck, placement of the femoral component in varus, incomplete seating of the femoral implant leading the stress concentration, and component malalignment leading to femoral neck impingement.
Mont et al. review the evolution of hip resurfacing arthroplasty. The authors discuss the advantages, disadvantages, and indications for the procedure.
While hip resurfacing boasts improved bone preservation both as a result of decreased bone removal and less proximal femoral stress shielding as compared to a total hip arthroplasty, the reliance on the femoral neck for stability results in as high as 4% risk for early post-operative femoral neck fracture. They note this is increased by factors which compromise proximal femoral bone stock, impair adequate exposure for ensuring optimal implant alignment, and increase stress at the bone-implant interface. They conclude that, while technically challenging, the procedure demonstrates excellent results in the young active patient.
Shimmin et al. evaluated risk factors for post-operative femoral neck fracture following hip resurfacing arthroplasty. The authors reported an overall incidence of 1.5% and found that risk was significantly increased among women. Furthermore, the majority of cases (85%) were associated with significant varus positioning of the femoral component (defined as >5 degrees), intra-operative notching of the superior aspect of the femoral neck, and technical issues including difficult exposure due to obesity, changes in intra-operative implant alignment, and incomplete seating of the femoral component during impaction. The authors concluded that restricted weight-bearing and close observation should be paramount in patients with
radiographic evidence of compromised stability, especially femoral notching.
Schmalzried et al. review the indications and outcomes of hip resurfacing arthroplasty. The authors note that careful patient selection is important, but that hip resurfacing reliably results in excellent functional outcomes among young active male patients with good bone stock. Revision is furthermore more straightforward and bone-preserving when converting to a THA. The authors advocate for select resurfacing despite current controversy.
Figure A is an immediate post-operative AP radiograph of the right hip demonstrating metal-on-metal hip resurfacing arthroplasty with evidence of femoral neck notching.
Figure B is a post-operative AP radiograph obtained at the time of follow-up demonstrating a complete displaced femoral neck fracture through the implant-bone interface.
Incorrect answers:
Answer 1: Due to the spectrum of disease, avascular necrosis in isolation is not a contraindication to hip resurfacing arthroplasty. However, avascular necrosis, specifically with cystic bone loss of the femoral head and neck, has been shown to be a risk factor for post-operative femoral neck fracture due to mechanical weakening of the bone-implant interface. The impact of bone viability remains unclear.
Answer 2: Notching of the femoral neck, entailing removal of cortical bone, has been found to have a strong association with post-operative femoral neck fracture.
Answer 3: Varus positioning of the femoral component or hip resurfacing performed in a varus hip have both been shown to increase the risk of postoperative femoral neck fracture. This is thought to be due to increased torsional stress at the bone-implant interface. Valgus positioning is not associated with increased failure rates.
Answer 4: Incomplete impaction or seating of the femoral component similarly has been found to result in stress-concentration at the implant-bone interface and is associated with an increased risk of post-operative femoral neck fracture.
-
-
When comparing conventional polyethylene liners to the newer highly cross-linked polyethylene liners, all of the following are true EXCEPT for one. Which of these statements about conventional poly liners is incorrect?
-
Higher steady-state femoral head penetration rate
-
More susceptible to adhesive wear
-
Generate smaller wear particle size
-
Increased fracture toughness
-
Increased tensile strength Corrent answer: 3
All of those statements regarding conventional polyethylene liners are true EXCEPT they generate larger (not smaller) wear particles than the highly cross-linked liners. It is the highly cross-linked liners that have been found to generate smaller wear particles.
Highly crossed-linked polyethylene in hip arthroplasty has been shown in multiple studies to be more wear resistant compared to conventional polyethylene, particularly in regards to adhesive wear. The highly cross-linked poly generates wear particles of a smaller size. In a randomized clinical study with 6.8 years follow-up, McCalden et al found that THA with highly cross-linked poly had a significantly lower femoral head penetration rate (0.003 mm/yr vs 0.051 mm/yr, p=0.025), which indicates less linear wear in the highly cross-linked poly. The disadvantage of highly cross-linked poly is that the cross-linking reduces its mechanical properties (e.g. decreased toughness, ductility, tensile strength, and fatigue strength) The Bradford reference looked at ex-planted liners and saw consistent evidence of early surface deformation and cracking; All explants exhibited some form of surface change, including surface cracking, abrasion, pitting, or scratching. The Jacobs paper makes the point that the cross-linked polys do seem to exhibit lower wear rates without significant osteolysis problems.
-
-
Patients display a Trendelenburg gait to compensate for weakness in which of the following muscle groups?
-
hip adductors
-
hip abductors
-
hip flexors
-
knee extensors
-
hip extensors
Corrent answer: 2
In the stance phase of gait in the hip of a normal hip individual, the hip joint force is produced by the body weight times its moment arm (distance from the
center of gravity of the body to the hip joint) which is balanced by the force exerted by the abductors. However, in individuals with hip joint problems, functional adaptations take place to reduce stimulus such as pain, muscle weakness or instability. Trendelenberg gait is one adaptation where avoidance of stress on the hip abductor muscles is achieved by shifting the body weight over the center of the hip joint, effectively decreasing the moment arm and requiring proportionally less effort from the abductors. This is shown in Illustration A. Therefore, a patient with a Trendelenberg lurch are effectively decreasing the hip joint forces as a method of adaptation.
-
-
A 73 year-old female underwent total knee arthroplasty 10 years ago. She sustained a proximal tibial shaft periprosthetic fracture after a ground level fall. Radiographs show that the fracture involves the tibial component's stem with loosening of the tibial component. Which of the following is the most appropriate treatment?
-
Open reduction and internal fixation of the tibia
-
External fixation
-
Intramedullary rod fixation
-
Revision with a long stem tibial component that bypasses the fracture
-
Fracture bracing
Corrent answer: 4
Revision with a long stem tibial component that bypasses the fracture is an appropriate treatment for tibial shaft fracture at the level of the implant with evidence of implant loosening.
Haidukewych et al present a Level 5 review that states a periprosthetic fracture that involves the stem of a component with evidence of loosening requires revision. This most common revision type is with a long stem, press fit component that bypasses the fracture by at least two cortical diameters.
Kim et al present a Level 5 review including a classification system and treatment algorithms. They state that Type II fractures are defined as those occurring with good bone stock and being reducible, but have loose or malpositioned components. They recommend that these fractures are treated by revision arthroplasty.
Incorrect Answers:
Answer 1: ORIF alone is the appropriate treatment for periprosthetic fractures without component loosening.
Answer 2: External fixation is not the most appropriate answer choice. Answer 3: IM fixation can be used for femoral periprosthetic fractures when the component is not loose.
Answer 5: Fracture bracing or casting is indicated for stable fractures with well fixed components.
-
-
All of the following interventions help restore anatomic limb length following total hip arthroplasty EXCEPT:
-
Preoperative templating
-
Use of an arthroplasty system incorporating variable neck lengths
-
Intraoperative assessment of limb length
-
Use of a modular arthroplasty system that allows variable femoral offset
-
Clinical and radiographic preoperative assessment for limb length discrepancy
-
Corrent answer: 4
Femoral offset can be decreased following total hip arthroplasty compared to the preoperative anatomy. A decrease in femoral offset moves the femur closer to the pelvis, which can result in impingement at the extremes of
movement. Moving the femur medially also results in soft tissue laxity. Both of these problems can cause instability and possible dislocation. Increasing femoral offset, which moves the femur laterally, will decrease impingement and improve soft tissue tension resulting in better stability without lengthening the leg.
Bourne et al review techniques to restore femoral offset during total hip arthroplasty. Pre-operative templating, intra-operative assessment of limb length, and high offset femoral necks are helpful to replicate femoral offset.
Suh et al reviewed a series of THA's to evaluate how well preoperative templating corresponded with postoperative limb lengthening and femoral offset. They reported both parameters were on average within 2 mm of the preoperative template.
Line A in Illustration A demonstrates femoral offset. Illustration B compares neck length and femoral offset affect on limb length.